A subgroup in Group theory












2














I am currently studying group theory, and its a quite new concept for me.
When learning about subgroups, I bumped into something.



In a problem where one wants to show that some set A is a group, is it sufficient enough to show that that set (A) satisfies all axioms of being a subgroup of a group B?



i.e.



If we know for sure that B is a group,
and we show that some set A satisfies all axioms of being a subgroup of a group B,
can we conclude that A is a group?










share|cite|improve this question


















  • 2




    Suppose set B is a group and A is a subset of B. If A is a subgroup, then A is also a group. So, yes.
    – Axion004
    2 days ago


















2














I am currently studying group theory, and its a quite new concept for me.
When learning about subgroups, I bumped into something.



In a problem where one wants to show that some set A is a group, is it sufficient enough to show that that set (A) satisfies all axioms of being a subgroup of a group B?



i.e.



If we know for sure that B is a group,
and we show that some set A satisfies all axioms of being a subgroup of a group B,
can we conclude that A is a group?










share|cite|improve this question


















  • 2




    Suppose set B is a group and A is a subset of B. If A is a subgroup, then A is also a group. So, yes.
    – Axion004
    2 days ago
















2












2








2


1





I am currently studying group theory, and its a quite new concept for me.
When learning about subgroups, I bumped into something.



In a problem where one wants to show that some set A is a group, is it sufficient enough to show that that set (A) satisfies all axioms of being a subgroup of a group B?



i.e.



If we know for sure that B is a group,
and we show that some set A satisfies all axioms of being a subgroup of a group B,
can we conclude that A is a group?










share|cite|improve this question













I am currently studying group theory, and its a quite new concept for me.
When learning about subgroups, I bumped into something.



In a problem where one wants to show that some set A is a group, is it sufficient enough to show that that set (A) satisfies all axioms of being a subgroup of a group B?



i.e.



If we know for sure that B is a group,
and we show that some set A satisfies all axioms of being a subgroup of a group B,
can we conclude that A is a group?







group-theory






share|cite|improve this question













share|cite|improve this question











share|cite|improve this question




share|cite|improve this question










asked 2 days ago









bladieblabladiebla

233




233








  • 2




    Suppose set B is a group and A is a subset of B. If A is a subgroup, then A is also a group. So, yes.
    – Axion004
    2 days ago
















  • 2




    Suppose set B is a group and A is a subset of B. If A is a subgroup, then A is also a group. So, yes.
    – Axion004
    2 days ago










2




2




Suppose set B is a group and A is a subset of B. If A is a subgroup, then A is also a group. So, yes.
– Axion004
2 days ago






Suppose set B is a group and A is a subset of B. If A is a subgroup, then A is also a group. So, yes.
– Axion004
2 days ago












3 Answers
3






active

oldest

votes


















4














It can be done easier. Let $G$ be a group and $U$ be a nonempty subset of $G$.
Then $U$ forms a subgroup of $G$ iff (1) for all $g,hin U$, $ghin U$, and (2) for each $gin U$, $g^{-1}in U$.



Then $U$ forms a subgroup of $G$ without checking the axioms point by point.



For instance, the unit element $ein G$ lies in $U$, since $U$ is nonempty and so $U$ has an element $g$. Then $g^{-1}in U$ by (2) and so by (1) $e= gg^{-1}in U$.






share|cite|improve this answer





















  • I'm sure you know this but for the OP: We can also collapse it down to just one condition: $U$ is a subgroup of $G$ $iff$ $forall (a in U) forall (b in U) ab^{-1} in U$ (this is often said verbally as "the subset is closed under subtraction", because in additive notation $ab^{-1}$ is denoted as $a-b$. And when you will study rings, you will switch to the additive notation for groups)
    – Ovi
    2 days ago












  • Surely, the ''subgroup criterion''.
    – Wuestenfux
    2 days ago










  • Yes ${}{}{}{}{}$
    – Ovi
    2 days ago



















1














No, as associativity may not be satisfied within the set. Consider, for instance, the structure $(S,*)$, with $S={1,a,b}$, and the following definition table for $*$:
$$
begin{matrix}
* & bf1 & bf{a} & bf{b} \
bf1 & 1 & a & b \
bf{a} & a & 1 & b \
bf{b} & b & a & 1
end{matrix}
$$

This satisfies the conditions for being a subgroup (it is closed under * and inverses, and it is nonempty), except for the fact that it is not in a group. But $(S,*)$ is not a group, because associativity fails: $1=a(ba) neq (ab)a=a$






share|cite|improve this answer










New contributor




pendermath is a new contributor to this site. Take care in asking for clarification, commenting, and answering.
Check out our Code of Conduct.














  • 1




    I don't see what this has to do with the question.
    – Matt Samuel
    2 days ago










  • The question reads: "...to show that some set A is a group, is it sufficient enough to show that that set (A) satisfies all axioms of being a subgroup of a group B...". I provide a counterexample to show that it is not enough. The structure (S,*) satisfies the conditions for a subgroup, but is not a group.
    – pendermath
    2 days ago












  • But if it's already a subset of a group with the same operation, it's automatically associative. We're not talking about an arbitrary operation.
    – Matt Samuel
    2 days ago










  • To quote the question, "If we know for sure that B is a group, and we show that some set A satisfies all axioms of being a subgroup of a group B, can we conclude that A is a group?"
    – Matt Samuel
    2 days ago










  • I am not assuming that is a subset of a group. Maybe I missed the question.
    – pendermath
    2 days ago



















0














If $G$ is a group and $Hsubseteq G$, then $H$ is a subgroup of $G$ iff it is nonvoid and closed under multiplication and inversion. You do not need to verify all of the axioms in this case because much is inherited from $G$.






share|cite|improve this answer





















    Your Answer





    StackExchange.ifUsing("editor", function () {
    return StackExchange.using("mathjaxEditing", function () {
    StackExchange.MarkdownEditor.creationCallbacks.add(function (editor, postfix) {
    StackExchange.mathjaxEditing.prepareWmdForMathJax(editor, postfix, [["$", "$"], ["\\(","\\)"]]);
    });
    });
    }, "mathjax-editing");

    StackExchange.ready(function() {
    var channelOptions = {
    tags: "".split(" "),
    id: "69"
    };
    initTagRenderer("".split(" "), "".split(" "), channelOptions);

    StackExchange.using("externalEditor", function() {
    // Have to fire editor after snippets, if snippets enabled
    if (StackExchange.settings.snippets.snippetsEnabled) {
    StackExchange.using("snippets", function() {
    createEditor();
    });
    }
    else {
    createEditor();
    }
    });

    function createEditor() {
    StackExchange.prepareEditor({
    heartbeatType: 'answer',
    autoActivateHeartbeat: false,
    convertImagesToLinks: true,
    noModals: true,
    showLowRepImageUploadWarning: true,
    reputationToPostImages: 10,
    bindNavPrevention: true,
    postfix: "",
    imageUploader: {
    brandingHtml: "Powered by u003ca class="icon-imgur-white" href="https://imgur.com/"u003eu003c/au003e",
    contentPolicyHtml: "User contributions licensed under u003ca href="https://creativecommons.org/licenses/by-sa/3.0/"u003ecc by-sa 3.0 with attribution requiredu003c/au003e u003ca href="https://stackoverflow.com/legal/content-policy"u003e(content policy)u003c/au003e",
    allowUrls: true
    },
    noCode: true, onDemand: true,
    discardSelector: ".discard-answer"
    ,immediatelyShowMarkdownHelp:true
    });


    }
    });














    draft saved

    draft discarded


















    StackExchange.ready(
    function () {
    StackExchange.openid.initPostLogin('.new-post-login', 'https%3a%2f%2fmath.stackexchange.com%2fquestions%2f3062766%2fa-subgroup-in-group-theory%23new-answer', 'question_page');
    }
    );

    Post as a guest















    Required, but never shown

























    3 Answers
    3






    active

    oldest

    votes








    3 Answers
    3






    active

    oldest

    votes









    active

    oldest

    votes






    active

    oldest

    votes









    4














    It can be done easier. Let $G$ be a group and $U$ be a nonempty subset of $G$.
    Then $U$ forms a subgroup of $G$ iff (1) for all $g,hin U$, $ghin U$, and (2) for each $gin U$, $g^{-1}in U$.



    Then $U$ forms a subgroup of $G$ without checking the axioms point by point.



    For instance, the unit element $ein G$ lies in $U$, since $U$ is nonempty and so $U$ has an element $g$. Then $g^{-1}in U$ by (2) and so by (1) $e= gg^{-1}in U$.






    share|cite|improve this answer





















    • I'm sure you know this but for the OP: We can also collapse it down to just one condition: $U$ is a subgroup of $G$ $iff$ $forall (a in U) forall (b in U) ab^{-1} in U$ (this is often said verbally as "the subset is closed under subtraction", because in additive notation $ab^{-1}$ is denoted as $a-b$. And when you will study rings, you will switch to the additive notation for groups)
      – Ovi
      2 days ago












    • Surely, the ''subgroup criterion''.
      – Wuestenfux
      2 days ago










    • Yes ${}{}{}{}{}$
      – Ovi
      2 days ago
















    4














    It can be done easier. Let $G$ be a group and $U$ be a nonempty subset of $G$.
    Then $U$ forms a subgroup of $G$ iff (1) for all $g,hin U$, $ghin U$, and (2) for each $gin U$, $g^{-1}in U$.



    Then $U$ forms a subgroup of $G$ without checking the axioms point by point.



    For instance, the unit element $ein G$ lies in $U$, since $U$ is nonempty and so $U$ has an element $g$. Then $g^{-1}in U$ by (2) and so by (1) $e= gg^{-1}in U$.






    share|cite|improve this answer





















    • I'm sure you know this but for the OP: We can also collapse it down to just one condition: $U$ is a subgroup of $G$ $iff$ $forall (a in U) forall (b in U) ab^{-1} in U$ (this is often said verbally as "the subset is closed under subtraction", because in additive notation $ab^{-1}$ is denoted as $a-b$. And when you will study rings, you will switch to the additive notation for groups)
      – Ovi
      2 days ago












    • Surely, the ''subgroup criterion''.
      – Wuestenfux
      2 days ago










    • Yes ${}{}{}{}{}$
      – Ovi
      2 days ago














    4












    4








    4






    It can be done easier. Let $G$ be a group and $U$ be a nonempty subset of $G$.
    Then $U$ forms a subgroup of $G$ iff (1) for all $g,hin U$, $ghin U$, and (2) for each $gin U$, $g^{-1}in U$.



    Then $U$ forms a subgroup of $G$ without checking the axioms point by point.



    For instance, the unit element $ein G$ lies in $U$, since $U$ is nonempty and so $U$ has an element $g$. Then $g^{-1}in U$ by (2) and so by (1) $e= gg^{-1}in U$.






    share|cite|improve this answer












    It can be done easier. Let $G$ be a group and $U$ be a nonempty subset of $G$.
    Then $U$ forms a subgroup of $G$ iff (1) for all $g,hin U$, $ghin U$, and (2) for each $gin U$, $g^{-1}in U$.



    Then $U$ forms a subgroup of $G$ without checking the axioms point by point.



    For instance, the unit element $ein G$ lies in $U$, since $U$ is nonempty and so $U$ has an element $g$. Then $g^{-1}in U$ by (2) and so by (1) $e= gg^{-1}in U$.







    share|cite|improve this answer












    share|cite|improve this answer



    share|cite|improve this answer










    answered 2 days ago









    WuestenfuxWuestenfux

    3,7061411




    3,7061411












    • I'm sure you know this but for the OP: We can also collapse it down to just one condition: $U$ is a subgroup of $G$ $iff$ $forall (a in U) forall (b in U) ab^{-1} in U$ (this is often said verbally as "the subset is closed under subtraction", because in additive notation $ab^{-1}$ is denoted as $a-b$. And when you will study rings, you will switch to the additive notation for groups)
      – Ovi
      2 days ago












    • Surely, the ''subgroup criterion''.
      – Wuestenfux
      2 days ago










    • Yes ${}{}{}{}{}$
      – Ovi
      2 days ago


















    • I'm sure you know this but for the OP: We can also collapse it down to just one condition: $U$ is a subgroup of $G$ $iff$ $forall (a in U) forall (b in U) ab^{-1} in U$ (this is often said verbally as "the subset is closed under subtraction", because in additive notation $ab^{-1}$ is denoted as $a-b$. And when you will study rings, you will switch to the additive notation for groups)
      – Ovi
      2 days ago












    • Surely, the ''subgroup criterion''.
      – Wuestenfux
      2 days ago










    • Yes ${}{}{}{}{}$
      – Ovi
      2 days ago
















    I'm sure you know this but for the OP: We can also collapse it down to just one condition: $U$ is a subgroup of $G$ $iff$ $forall (a in U) forall (b in U) ab^{-1} in U$ (this is often said verbally as "the subset is closed under subtraction", because in additive notation $ab^{-1}$ is denoted as $a-b$. And when you will study rings, you will switch to the additive notation for groups)
    – Ovi
    2 days ago






    I'm sure you know this but for the OP: We can also collapse it down to just one condition: $U$ is a subgroup of $G$ $iff$ $forall (a in U) forall (b in U) ab^{-1} in U$ (this is often said verbally as "the subset is closed under subtraction", because in additive notation $ab^{-1}$ is denoted as $a-b$. And when you will study rings, you will switch to the additive notation for groups)
    – Ovi
    2 days ago














    Surely, the ''subgroup criterion''.
    – Wuestenfux
    2 days ago




    Surely, the ''subgroup criterion''.
    – Wuestenfux
    2 days ago












    Yes ${}{}{}{}{}$
    – Ovi
    2 days ago




    Yes ${}{}{}{}{}$
    – Ovi
    2 days ago











    1














    No, as associativity may not be satisfied within the set. Consider, for instance, the structure $(S,*)$, with $S={1,a,b}$, and the following definition table for $*$:
    $$
    begin{matrix}
    * & bf1 & bf{a} & bf{b} \
    bf1 & 1 & a & b \
    bf{a} & a & 1 & b \
    bf{b} & b & a & 1
    end{matrix}
    $$

    This satisfies the conditions for being a subgroup (it is closed under * and inverses, and it is nonempty), except for the fact that it is not in a group. But $(S,*)$ is not a group, because associativity fails: $1=a(ba) neq (ab)a=a$






    share|cite|improve this answer










    New contributor




    pendermath is a new contributor to this site. Take care in asking for clarification, commenting, and answering.
    Check out our Code of Conduct.














    • 1




      I don't see what this has to do with the question.
      – Matt Samuel
      2 days ago










    • The question reads: "...to show that some set A is a group, is it sufficient enough to show that that set (A) satisfies all axioms of being a subgroup of a group B...". I provide a counterexample to show that it is not enough. The structure (S,*) satisfies the conditions for a subgroup, but is not a group.
      – pendermath
      2 days ago












    • But if it's already a subset of a group with the same operation, it's automatically associative. We're not talking about an arbitrary operation.
      – Matt Samuel
      2 days ago










    • To quote the question, "If we know for sure that B is a group, and we show that some set A satisfies all axioms of being a subgroup of a group B, can we conclude that A is a group?"
      – Matt Samuel
      2 days ago










    • I am not assuming that is a subset of a group. Maybe I missed the question.
      – pendermath
      2 days ago
















    1














    No, as associativity may not be satisfied within the set. Consider, for instance, the structure $(S,*)$, with $S={1,a,b}$, and the following definition table for $*$:
    $$
    begin{matrix}
    * & bf1 & bf{a} & bf{b} \
    bf1 & 1 & a & b \
    bf{a} & a & 1 & b \
    bf{b} & b & a & 1
    end{matrix}
    $$

    This satisfies the conditions for being a subgroup (it is closed under * and inverses, and it is nonempty), except for the fact that it is not in a group. But $(S,*)$ is not a group, because associativity fails: $1=a(ba) neq (ab)a=a$






    share|cite|improve this answer










    New contributor




    pendermath is a new contributor to this site. Take care in asking for clarification, commenting, and answering.
    Check out our Code of Conduct.














    • 1




      I don't see what this has to do with the question.
      – Matt Samuel
      2 days ago










    • The question reads: "...to show that some set A is a group, is it sufficient enough to show that that set (A) satisfies all axioms of being a subgroup of a group B...". I provide a counterexample to show that it is not enough. The structure (S,*) satisfies the conditions for a subgroup, but is not a group.
      – pendermath
      2 days ago












    • But if it's already a subset of a group with the same operation, it's automatically associative. We're not talking about an arbitrary operation.
      – Matt Samuel
      2 days ago










    • To quote the question, "If we know for sure that B is a group, and we show that some set A satisfies all axioms of being a subgroup of a group B, can we conclude that A is a group?"
      – Matt Samuel
      2 days ago










    • I am not assuming that is a subset of a group. Maybe I missed the question.
      – pendermath
      2 days ago














    1












    1








    1






    No, as associativity may not be satisfied within the set. Consider, for instance, the structure $(S,*)$, with $S={1,a,b}$, and the following definition table for $*$:
    $$
    begin{matrix}
    * & bf1 & bf{a} & bf{b} \
    bf1 & 1 & a & b \
    bf{a} & a & 1 & b \
    bf{b} & b & a & 1
    end{matrix}
    $$

    This satisfies the conditions for being a subgroup (it is closed under * and inverses, and it is nonempty), except for the fact that it is not in a group. But $(S,*)$ is not a group, because associativity fails: $1=a(ba) neq (ab)a=a$






    share|cite|improve this answer










    New contributor




    pendermath is a new contributor to this site. Take care in asking for clarification, commenting, and answering.
    Check out our Code of Conduct.









    No, as associativity may not be satisfied within the set. Consider, for instance, the structure $(S,*)$, with $S={1,a,b}$, and the following definition table for $*$:
    $$
    begin{matrix}
    * & bf1 & bf{a} & bf{b} \
    bf1 & 1 & a & b \
    bf{a} & a & 1 & b \
    bf{b} & b & a & 1
    end{matrix}
    $$

    This satisfies the conditions for being a subgroup (it is closed under * and inverses, and it is nonempty), except for the fact that it is not in a group. But $(S,*)$ is not a group, because associativity fails: $1=a(ba) neq (ab)a=a$







    share|cite|improve this answer










    New contributor




    pendermath is a new contributor to this site. Take care in asking for clarification, commenting, and answering.
    Check out our Code of Conduct.









    share|cite|improve this answer



    share|cite|improve this answer








    edited 2 days ago





















    New contributor




    pendermath is a new contributor to this site. Take care in asking for clarification, commenting, and answering.
    Check out our Code of Conduct.









    answered 2 days ago









    pendermathpendermath

    16010




    16010




    New contributor




    pendermath is a new contributor to this site. Take care in asking for clarification, commenting, and answering.
    Check out our Code of Conduct.





    New contributor





    pendermath is a new contributor to this site. Take care in asking for clarification, commenting, and answering.
    Check out our Code of Conduct.






    pendermath is a new contributor to this site. Take care in asking for clarification, commenting, and answering.
    Check out our Code of Conduct.








    • 1




      I don't see what this has to do with the question.
      – Matt Samuel
      2 days ago










    • The question reads: "...to show that some set A is a group, is it sufficient enough to show that that set (A) satisfies all axioms of being a subgroup of a group B...". I provide a counterexample to show that it is not enough. The structure (S,*) satisfies the conditions for a subgroup, but is not a group.
      – pendermath
      2 days ago












    • But if it's already a subset of a group with the same operation, it's automatically associative. We're not talking about an arbitrary operation.
      – Matt Samuel
      2 days ago










    • To quote the question, "If we know for sure that B is a group, and we show that some set A satisfies all axioms of being a subgroup of a group B, can we conclude that A is a group?"
      – Matt Samuel
      2 days ago










    • I am not assuming that is a subset of a group. Maybe I missed the question.
      – pendermath
      2 days ago














    • 1




      I don't see what this has to do with the question.
      – Matt Samuel
      2 days ago










    • The question reads: "...to show that some set A is a group, is it sufficient enough to show that that set (A) satisfies all axioms of being a subgroup of a group B...". I provide a counterexample to show that it is not enough. The structure (S,*) satisfies the conditions for a subgroup, but is not a group.
      – pendermath
      2 days ago












    • But if it's already a subset of a group with the same operation, it's automatically associative. We're not talking about an arbitrary operation.
      – Matt Samuel
      2 days ago










    • To quote the question, "If we know for sure that B is a group, and we show that some set A satisfies all axioms of being a subgroup of a group B, can we conclude that A is a group?"
      – Matt Samuel
      2 days ago










    • I am not assuming that is a subset of a group. Maybe I missed the question.
      – pendermath
      2 days ago








    1




    1




    I don't see what this has to do with the question.
    – Matt Samuel
    2 days ago




    I don't see what this has to do with the question.
    – Matt Samuel
    2 days ago












    The question reads: "...to show that some set A is a group, is it sufficient enough to show that that set (A) satisfies all axioms of being a subgroup of a group B...". I provide a counterexample to show that it is not enough. The structure (S,*) satisfies the conditions for a subgroup, but is not a group.
    – pendermath
    2 days ago






    The question reads: "...to show that some set A is a group, is it sufficient enough to show that that set (A) satisfies all axioms of being a subgroup of a group B...". I provide a counterexample to show that it is not enough. The structure (S,*) satisfies the conditions for a subgroup, but is not a group.
    – pendermath
    2 days ago














    But if it's already a subset of a group with the same operation, it's automatically associative. We're not talking about an arbitrary operation.
    – Matt Samuel
    2 days ago




    But if it's already a subset of a group with the same operation, it's automatically associative. We're not talking about an arbitrary operation.
    – Matt Samuel
    2 days ago












    To quote the question, "If we know for sure that B is a group, and we show that some set A satisfies all axioms of being a subgroup of a group B, can we conclude that A is a group?"
    – Matt Samuel
    2 days ago




    To quote the question, "If we know for sure that B is a group, and we show that some set A satisfies all axioms of being a subgroup of a group B, can we conclude that A is a group?"
    – Matt Samuel
    2 days ago












    I am not assuming that is a subset of a group. Maybe I missed the question.
    – pendermath
    2 days ago




    I am not assuming that is a subset of a group. Maybe I missed the question.
    – pendermath
    2 days ago











    0














    If $G$ is a group and $Hsubseteq G$, then $H$ is a subgroup of $G$ iff it is nonvoid and closed under multiplication and inversion. You do not need to verify all of the axioms in this case because much is inherited from $G$.






    share|cite|improve this answer


























      0














      If $G$ is a group and $Hsubseteq G$, then $H$ is a subgroup of $G$ iff it is nonvoid and closed under multiplication and inversion. You do not need to verify all of the axioms in this case because much is inherited from $G$.






      share|cite|improve this answer
























        0












        0








        0






        If $G$ is a group and $Hsubseteq G$, then $H$ is a subgroup of $G$ iff it is nonvoid and closed under multiplication and inversion. You do not need to verify all of the axioms in this case because much is inherited from $G$.






        share|cite|improve this answer












        If $G$ is a group and $Hsubseteq G$, then $H$ is a subgroup of $G$ iff it is nonvoid and closed under multiplication and inversion. You do not need to verify all of the axioms in this case because much is inherited from $G$.







        share|cite|improve this answer












        share|cite|improve this answer



        share|cite|improve this answer










        answered 2 days ago









        ncmathsadistncmathsadist

        42.4k259102




        42.4k259102






























            draft saved

            draft discarded




















































            Thanks for contributing an answer to Mathematics Stack Exchange!


            • Please be sure to answer the question. Provide details and share your research!

            But avoid



            • Asking for help, clarification, or responding to other answers.

            • Making statements based on opinion; back them up with references or personal experience.


            Use MathJax to format equations. MathJax reference.


            To learn more, see our tips on writing great answers.





            Some of your past answers have not been well-received, and you're in danger of being blocked from answering.


            Please pay close attention to the following guidance:


            • Please be sure to answer the question. Provide details and share your research!

            But avoid



            • Asking for help, clarification, or responding to other answers.

            • Making statements based on opinion; back them up with references or personal experience.


            To learn more, see our tips on writing great answers.




            draft saved


            draft discarded














            StackExchange.ready(
            function () {
            StackExchange.openid.initPostLogin('.new-post-login', 'https%3a%2f%2fmath.stackexchange.com%2fquestions%2f3062766%2fa-subgroup-in-group-theory%23new-answer', 'question_page');
            }
            );

            Post as a guest















            Required, but never shown





















































            Required, but never shown














            Required, but never shown












            Required, but never shown







            Required, but never shown

































            Required, but never shown














            Required, but never shown












            Required, but never shown







            Required, but never shown







            Popular posts from this blog

            Mario Kart Wii

            What does “Dominus providebit” mean?

            Antonio Litta Visconti Arese